Bleibt das 4343\frac{4}{3} Problem des klassischen Elektromagnetismus in der Quantenmechanik?

In Band II Kapitel 28 der Feymann Lectures on Physics diskutiert Feynman das Berüchtigte 4 3 Problem des klassischen Elektromagnetismus. Angenommen, Sie haben ein geladenes Teilchen mit Radius a und aufladen q (gleichmäßig auf der Oberfläche verteilt). Wenn Sie die Energiedichte des elektromagnetischen Feldes über den gesamten Raum außerhalb des Teilchens integrieren, erhalten Sie die gesamte elektromagnetische Energie, die ein Ausdruck ist, der proportional zu ist c 2 . Die Energie geteilt durch c 2 ist das, was wir normalerweise die Masse nennen. Wenn wir also die "elektromagnetische Masse" auf diese Weise berechnen, erhalten wir m = 1 2 1 4 π ϵ 0 q 2 a c 2 . Nimmt man hingegen die Impulsdichte des elektromagnetischen Feldes und integriert sie über den gesamten Raum außerhalb des Teilchens, erhält man den gesamten elektromagnetischen Impuls, der sich herausstellt (z v << c ) proportional zur Geschwindigkeit des Teilchens. Die Konstante der Proportionalität von Impuls und Geschwindigkeit ist das, was wir Masse nennen. Wenn wir also die elektromagnetische Masse auf diese Weise berechnen würden, würden wir sie erhalten m = 2 3 1 4 π ϵ 0 q 2 a c 2 , welches ist 4 3 mal den Wert, den wir vorher hatten! Das ist die 4 3 Problem.

Feynman behauptet, dass dieses grundlegende Problem bestehen bleibt, wenn wir zur Quantenelektrodynamik übergehen. Hatte er recht, und wenn ja, hat sich die Situation seitdem geändert? 1960 ' s als er schrieb? Ich habe Behauptungen im Internet gesehen (ich habe die Links nicht), dass die 4 3 Problem ist immer noch da in QED, aber statt 4 3 der Koeffizient ist etwas näher an 1. Stimmt das, und wenn ja, wie lautet der Koeffizient? All dies hängt natürlich mit Fragen der Selbstenergie und der Renormalisierung zusammen.

Jede Hilfe wäre sehr willkommen.

Antworten (4)

Problemskizze

In der zitierten Referenz beginnt Feynman seine Argumentation mit der Feststellung, dass die Energie für eine Trägheitskugel einen Radius hat a und einheitliche Ladung q wird von gegeben

U e l e c = R 3 d v u e l e c = R 3 d v ( ϵ 0 E 2 ) = 1 2 a q 2 4 π ϵ 0 e 2 2 a ( )

wo E und u e l e c sind sein elektrisches Feld und seine Dichte und die letzte Gleichung definiert das Symbol e 2 . Betrachten wir nun die Kugel als sich mit konstanter Geschwindigkeit bewegend v c , seine Impulsdichte ergibt sich aus dem Poynting-Vektor:

p = R 3 d v S = R 3 d v ( ϵ 0 E × B ) = ( 2 3 e 2 a c 2 ) v m e v , ( )

wobei die letzte Gleichung die elektromagnetische Masse definiert m e . Wir können auch eine zweite elektrische Masse zuordnen m e ' zum Feld U e l e c unter Verwendung der speziellen Relativitätstheorie (SR):

E = m c 2 U e l e c c 2 = m e ' e 2 2 a c 2 = m e ' = 3 4 m e U e l e c = 4 3 m e c 2 ,

das heißt, die "relativistische" Masse m e ' ist nicht dasselbe wie die "elektrische" Masse m e , wobei das Absurde in den obigen Gleichungen explizit ist.

Das Schlupfloch

Haben wir das Recht, SR in einem einzigen Schritt einer Ableitung aufzurufen? Obwohl wir Einsteins berühmte Formel verwendet haben, um zu dem zu gelangen 4 3 Problem, beachten Sie, dass wir SR früher nicht verwendet haben: ( ) und ( ) sind nicht einmal Lorentz-invariant! Wenn wir die Berechnungen anpassen, um SR einzubeziehen, U e l e c und p verwandeln als

U e l e c ' = γ ( U e l e c + p v ) p ' = γ ( p + U e l e c v c 2 ) + ( γ 1 ) v × ( v × p ) .

Das heißt, für ein ruhendes Teilchen haben wir p = 0 und Einführung von Geschwindigkeit auf relativistisch sinnvolle Weise durch Wechsel in ein sich bewegendes Bezugssystem mit Geschwindigkeit v ,

U e l e c ' = γ U e l e c = γ ( e 2 2 a ) = γ m e ' c 2 p ' = γ ( U e l e c c 2 ) v = γ ( e 2 2 a c 2 ) v = γ m e ' v ,

wo jetzt, siehe da, alles passt und es absolut keine gibt 4 3 Paradox. Es ging nur darum, sich korrekt zwischen Referenzrahmen zu bewegen.

Was meinte Feynman?

Als einer der brillantesten Physiker der Geschichte würde Richard Feynman SR niemals so missbrauchen, wie es notwendig ist, um zu dem zu gelangen 4 3 Paradox. Tatsächlich taten Abraham und Lorentz, die Entdecker, dies nur, weil sie nicht wirklich wussten, wie man zwischen Referenzrahmen transformiert, da SR zu diesem Zeitpunkt noch nicht fertig war. Lassen Sie uns dann zitieren, was Feynman wirklich gesagt hat:

Mit den Ideen der Maxwellschen Theorie sind Schwierigkeiten verbunden, die von der Quantenmechanik nicht gelöst werden und nicht direkt damit verbunden sind. Sie können sagen: „Vielleicht ist es sinnlos, sich über diese Schwierigkeiten Sorgen zu machen. Da die Quantenmechanik die Gesetze der Elektrodynamik ändern wird, sollten wir abwarten, welche Schwierigkeiten es nach der Modifikation gibt.“ Wenn jedoch der Elektromagnetismus mit der Quantenmechanik verbunden wird, bleiben die Schwierigkeiten bestehen.

In der Tat, die 4 3 Dieses Problem wird nicht durch Quantenmechanik (QM) gelöst, sondern durch den sorgfältigen Einsatz von SR. Abraham und Lorentz verwendet ( ) und ( ) wahllos, weil sie nicht wussten, dass sich bewegende Elektronen laut SR zu Ellipsoiden verformt werden. Der schöne Artikel von Rohrlich , dessen Zusammenfassung diese Antwort ist, zeigt, dass der Fehler bei der Vernachlässigung der Deformation des Elektrons genau gleich ist 1 3 m e ' c 2 , was ihr Ergebnis korrigieren und auch die loswerden würde 4 3 Problem. Ich glaube, Feynman sprach von anderen Problemen, die von der klassischen zur Quantenelektrodynamik (QED) wandern, nicht von diesem scheinbaren Paradoxon. Das Problem der Selbstenergie erfordert zum Beispiel, dass das Elektron in der QED entweder eine eher nicht intuitive interne Struktur hat oder mit Dingen mit negativer Masse interagiert, sonst würde es explodieren (abgesehen von der Renormalisierung). Dieses Problem wandert aus der klassischen Elektrodynamik fälschlicherweise in die Postulation einer „Poincaré-Spannung“ ein, die das Elektron nicht nur stabilisieren, sondern auch korrigieren würde 1 3 Faktor fehlt in der 4 3 Problem. Glücklicherweise halfen viele Physiker dabei, das Problem der elektronischen Stabilität von der zu trennen 4 3 Problem, was nicht wirklich ein Problem ist, da es in QED nicht wirklich auftaucht. Das Problem der inneren Struktur des Elektrons ist jedoch ziemlich real.

„Haben wir das Recht, SR in einem einzigen Schritt einer Ableitung aufzurufen? Obwohl wir Einsteins berühmte Formel verwendet haben, um zu dem 43-Problem zu gelangen, beachten Sie, dass wir SR nicht früher verwendet haben: (∗) und (∗∗) sind nicht einmal Lorentz-invariant !" Tatsächlich berücksichtigt Feynman das Verformungsproblem vollständig, löst jedoch nicht das 4/3-Problem. Scrollen Sie nach unten zu der Stelle, an der er sagt: "Lassen Sie uns unsere elektromagnetische Theorie der Masse weiterverfolgen. Unsere Berechnung war für v≪c; was passiert, wenn wir zu hohen Geschwindigkeiten gehen?" feynmanlectures.caltech.edu/II_28.html Sie erhalten eine Lorentz-invariante Form, aber sie hat immer noch eine 2323 Faktor.
Erstens gilt Feynmans Formel 28.7 für die relativistische Änderung der falschen Masse, m eme , die in SR nicht vorkommt: Der korrekte Begriff ist m em'e . Zweitens bedeutet die vollständige Berücksichtigung der Elektronendeformation, dass ein Integral nicht vernachlässigt wird, das nur innerhalb eines nackten Elektrons null ist. Ich fordere Sie dringend auf, Rohrlichs Artikel zu lesen, in dem alles sehr gut dargestellt ist, insbesondere der letzte Abschnitt, in dem er nach dem Spannungs-Energie-Tensor auf der Innenseite und Außenseite eines ellipsoidischen Elektrons auflöst.
Ich weiß nicht, was Sie meinen mit "Feynmans Formel 28.7 ist für die relativistische Änderung der falschen Masse, m eme , die in SR nicht auftritt". Formel 28.7 berechnet nur den relativistischen Impuls, es geht noch nicht um elektromagnetische Masse.
Prüfen Sie genau: Feynman definiert die elektromagnetische Masse m eme in 28,4, mit dem Faktor 2/3 davor. Dann verwendet er es später in 28.7, um das relativistische Momentum zu definieren. Dies ist jedoch nicht richtig, und Feynman arbeitet nur die Konsequenzen einer schlechten Wahl aus. Die richtige, richtige Masse kommt von SR und hat keinen 2/3 Faktor davor, da sie gegeben ist durch m e = U e l e c / c 2m'e=Ue l e c/c2 . Diese Masse wird dann später verwendet, um den relativistisch korrekten Impuls p = γ m e v zu erhaltenp⃗ '= γm'ev⃗  . Die Masse in 28.4 ist falsch , da sie von einem Ausdruck stammt, der in SR nicht korrekt ist.
Er leitet 28,7 nicht aus 28,4 ab. Er leitet zuerst 28,4 aus 28,3 ab, dann liefert er 28,7 als relativistisches Analogon der Berechnung, die durchgeführt wurde, um 28,3 zu erhalten, um zu zeigen, dass 28,4 aus 28,7 auf die gleiche Weise stammt wie aus 28,3.
Ich bekräftige, was ich zuvor gesagt habe: Die richtige Masse, die auftaucht, wenn man den richtigen relativistischen Impuls verwendet, ist m em'e , nicht ich eme . Dies wird explizit in meinen Berechnungen durchgeführt. Feynmans Formel 28.7 ist falsch .
Das kann bei Formel 28.7 unmöglich ein Problem sein, denn bei Formel 28.7 geht es überhaupt nicht um elektromagnetische Masse. Es geht um elektromagnetisches Moment, und dann wird 28,7 verwendet, um die elektromagnetische Masse zu erhalten.
Aufgabe 1: Erreiche Formel 28.7, ohne von einem Anfangsimpuls gleich m e v auszugehenmev⃗  . Hinweis: Es ist unmöglich. Aufgabe 2: Definiere die Masse mit m e = U e l e c / c 2m'e=Ue l e c/c2 , wiederholen Sie dann die Berechnungen, die ich angezeigt habe, und erreichen Sie den richtigen Impuls im kleinen Geschwindigkeitsbereich m e vm'ev⃗  .
Feynman beschreibt in Umrissen, wie es gemacht wird, und es ist nicht die Verwendung von Formel 28.4. Durch Integration der Impulsdichte unter Berücksichtigung der Relativitätstheorie: „Frühe Versuche führten zu einer gewissen Verwirrung, aber Lorentz erkannte, dass sich die geladene Kugel bei hohen Geschwindigkeiten zu einem Ellipsoid zusammenziehen würde und dass sich die Felder gemäß den Formeln (26.6 ) und (26.7) haben wir für den relativistischen Fall in Kapitel 26 hergeleitet. Wenn Sie in diesem Fall die Integrale für p durchführen ...“ Ich bezweifle sehr, dass Feynman bluffte, er hat vermutlich die entsprechende Berechnung durchgeführt.
Ich lasse Sie die Übungen machen, die ich vorgeschlagen habe. Dann suchen Sie bitte den Fehler in: for p = 0p⃗ = 0 , m ' e = U e l e c / c 2p ' = γ U e l e c v / c 2 = γ m ' e vm'e=Ue l e c/c2p⃗ '= γUe l e cv⃗ /c2= γm'ev⃗  . Diese Berechnung wurde von Schwinger, Fermi, Rohrlich und Dutzenden anderen durchgeführt. Ohne meine Antwort richtig zu verstehen oder die von mir bereitgestellten Referenzen zu lesen, hat es keinen Sinn, diese Diskussion fortzusetzen.
Warum tue ich das nicht: Ich kann eine neue Frage auf der Seite posten, die fragt, wie das Integral für Formel 28.7 gemacht wird. Wie Sie sagten, ist Feynman "einer der brillantesten Physiker der Geschichte", also sollten wir nicht abtun, was er über die Integration sagt, um p zu findenp Aus der Hand.
Sie können das fragen, aber es ist trivial: Um zu 28,7 zu ​​gelangen, ging Feynman von 28,3 aus und verwendete später das Transformationsgesetz p = γ pp⃗ '= γp⃗  vom Anfangsimpuls starten m e vmev⃗  und bilde es ab auf γ m e vγmev⃗  . Gleichung 28.7 ist offensichtlich nicht relativistisch korrekt, da die 2 / 32/3 _ _ Faktor davor verhindert, dass sich der zugehörige 4-Impuls in einen 4-Vektor umwandelt. Gleichung 28.7 verwendet also das richtige Transformationsgesetz für den falschen Impuls, den man aus einer relativistisch falschen Gleichung erhält. Die richtige ergibt sich aus p = m e vp⃗ =m'ev⃗  .
„um zu 28.7 zu gelangen, ging Feynman von 28.3 aus und benutzte später das Transformationsgesetz p = γ p von der Anfangsdynamik ausgehen m e v und bilde es auf γ m e v ab .“ Das ist nicht das, was er getan hat. Er sagt, dass er die Impulsdichte integriert hat, indem er die Feldtransformationen aus den Formeln 26.6 und 26.7 verwendet.

In der modernen Sichtweise der Quantenfeldtheorie ist die 4 / 3 Problem sowie damit verbundene Fragen zu Punktgebühren sind Fragen der Regularisierung.

Das Problem ist, dass Sie es mit einer singulären Größe zu tun haben wollen, nämlich der Energie und dem Impuls einer idealen Punktladung, aber es ist unmöglich, etwas Bestimmtes zu sagen, weil alle diese Größen divergieren. Um eine endliche Antwort zu erhalten, modifizieren Sie das System, indem Sie es auf irgendeine Weise regulieren, sodass die Antworten endlich herauskommen. Im Allgemeinen kann und wird die Regularisierung Symmetrien brechen, und wir versuchen im Allgemeinen unser Bestes, um ein Regularisierungsschema zu wählen, das die Symmetrien bewahrt, die uns am wichtigsten sind.

Der philosophische Grund dafür ist, dass wir wissen, dass unsere Theorien nur bis zu einer bestimmten Grenzskala gültig sind Λ , über der neue Phänomene auftreten. Unterhalb des Grenzwerts können die Auswirkungen dieser neuen Phänomene in Form einer effektiven Lagrange-Funktion parametrisiert werden. Daher können wir die gleichen Niederenergievorhersagen in jedem Regularisierungsschema erhalten, das den gleichen effektiven Lagrange ergibt, selbst wenn diese Schemata eine dramatisch unterschiedliche Physik über die Grenzskala bringen.

In diesem Fall regularisiert Feynman die klassische Theorie, indem er die Feldintegrale bei abschneidet Λ 1 / a . Leider bewahrt dieses Regularisierungsschema die Lorentz-Invarianz nicht, und außerdem wird die Lorentz-Invarianz nicht einmal in der Kontinuumsgrenze wiederhergestellt Λ . Das ist keine Seltenheit. Auch die Gitter-QCD muss sich mit diesem Problem befassen, da eine Gitterregularisierung die Lorentz-Invarianz eindeutig bricht.

Der Grund, warum Gitter-QCD in der RG-Sprache trotzdem funktioniert, ist, dass ihre Systeme so konstruiert sind, dass nur zusätzliche irrelevante Operatoren erzeugt werden können, deren Effekte in der Kontinuumsgrenze verschwinden a 0 . Intuitiv besteht das Problem mit Feynmans Regularisierung darin, dass sie zusätzliche relevante Operatoren zulässt, die die effektive Lagrange-Funktion auf jeden Fall durcheinander bringen a ist, obwohl diese Sprache nicht exakt ist, da Feynman sich nicht mit einer Quantenfeldtheorie befasst.

Die moderne Auflösung des 4 / 3 Problem in der Quantenfeldtheorie ist im Wesentlichen per Befehl. Eine nicht regulierte Theorie ist schlecht definiert; wir können nicht von unregulierter QED sprechen, weil eine solche Theorie nicht einmal existiert. Stattdessen müssen wir von Anfang an eine Regulierungsbehörde im Auge behalten. Da wir wissen, dass die Lorentz-Invarianz eine Symmetrie unserer Welt ist, wählen wir einen Regulator, der die Lorentz-Invarianz in der Kontinuumsgrenze wiederherstellt.

Mir ist klar, wie das Problem ursprünglich aus dem Verständnis der Physik punktförmig geladener Teilchen entstand, und das Ersetzen des punktförmig geladenen Teilchens durch eine geladene Hülle ist eine sehr physikalische Methode, um nur die Cut-Off-Regularisierung zu setzen, die die Lorentz-Symmetrie bricht. Man kann jedoch die wichtigere Frage der Physik punktförmiger geladener Teilchen vergessen und sich einfach für die Physik einer geladenen Hülle an sich interessieren. Wenn man das tut, denke ich das 4 / 3 Problem bleibt eine berechtigte Frage, richtig?

Neben H. Poincaré war E. Fermi der erste Physiker, der eine korrekte relativistische "Ableitung" durchführte.

Hier möchte ich mich auf den physikalischen Effekt der berühmten Selbstaktion konzentrieren: Es ist einfach eine Selbstinduktion, die jeder Beschleunigung (Zeitänderung eines konstanten Stroms) "verlangsamt" oder "widersteht". Anscheinend ist es kein gewünschter Effekt - es ist überhaupt kein kleiner Strahlungswiderstand, unabhängig von der Größe des Elektrons.

Beachten Sie, dass sich die EMF gut mit dem Elektron "bewegt" - gemäß der Maxwell-Gleichung, sodass sie in den (mechanischen) Elektronengleichungen nicht "erneut" berücksichtigt werden muss. Der Strahlungswiderstand muss eher aus unterschiedlichen physikalischen Vorstellungen erraten werden als aus „Eigenaktion“.

In der QED bleibt dieses Problem bestehen und wird "gelöst", indem die (falschen) Gleichungslösungen modifiziert werden, anstatt die falschen Gleichungen zu modifizieren.

Meine Forschungsergebnisse befinden sich noch in einem embryonalen Stadium, aber ich gehe von einer anderen physikalischen Idee aus, die den Strahlungswiderstand gut einfängt.

Ich stimme zu: "In QED bleibt dieses Problem bestehen und es wird "gelöst", indem die (falschen) Gleichungslösungen geändert werden, anstatt die falschen Gleichungen zu ändern". Leider ist diese Antwort weitgehend unverständlich.
@QuantumBrick: Schade. Ich habe einen weiteren Hinweis auf Rohlich: Fritz Rohrlich, The dynamics of a chargedarticle, (2008) arxiv.org/abs/0804.4614 , wo er feststellt, dass das Problem in CED noch nicht (auf zufriedenstellende Weise) gelöst ist.

In Bezug auf das in Band II, Kapitel 28 der Feynman Lectures on Physics beschriebene Problem besteht der 4/3-Problemgrund darin, dass einer der Terme, die die Bewegung des Teilchens beeinflussen, nicht berücksichtigt wird. Dieser Begriff ist die Zerstörung der Feldenergie an der Vorderseite der geladenen Kugel und ihre Erzeugung an der Rückseite. Während sich die Gesamtfeldenergie nicht ändert, können Sie, wenn Sie die Energie lokal auswerten, sehen, dass der v·E-Term vorne bedeutet, dass die Energie vom Feld zum Teilchen gebracht wird, und hinten das Gegenteil passiert, da dies ständig geschieht, kommt seine Wirkung einer Rückwärtsbewegung gleich. Wenn Sie es zu der normalen Bewegung hinzufügen, die durch den Impuls verursacht wird, stellen Sie fest, dass Sie eine Masse m_e = (1/2)e^2/a benötigen, und es gibt keine Inkonsistenzen mehr.

Ich habe hier ein kleines Dokument zum Thema eingestellt: https://www.slideshare.net/SergioPL81/adding-a-shift-term-to-solve-the-43-problem-in-classical-electrodinamics